Learn more

Refine search


Results (14 matches)

  displayed columns for results
Label Class Conductor Rank Torsion CM Regulator Weierstrass coefficients Weierstrass equation mod-$m$ images
4008.b1 4008.b \( 2^{3} \cdot 3 \cdot 167 \) $1$ $\mathsf{trivial}$ $3.426139440$ $[0, 1, 0, -296, -2064]$ \(y^2=x^3+x^2-296x-2064\) 1336.2.0.?
8016.c1 8016.c \( 2^{4} \cdot 3 \cdot 167 \) $1$ $\mathsf{trivial}$ $0.178421141$ $[0, -1, 0, -296, 2064]$ \(y^2=x^3-x^2-296x+2064\) 1336.2.0.?
12024.d1 12024.d \( 2^{3} \cdot 3^{2} \cdot 167 \) $1$ $\mathsf{trivial}$ $1.729599078$ $[0, 0, 0, -2667, 53062]$ \(y^2=x^3-2667x+53062\) 1336.2.0.?
24048.h1 24048.h \( 2^{4} \cdot 3^{2} \cdot 167 \) $1$ $\mathsf{trivial}$ $4.586362635$ $[0, 0, 0, -2667, -53062]$ \(y^2=x^3-2667x-53062\) 1336.2.0.?
32064.i1 32064.i \( 2^{6} \cdot 3 \cdot 167 \) $0$ $\mathsf{trivial}$ $1$ $[0, -1, 0, -1185, -15327]$ \(y^2=x^3-x^2-1185x-15327\) 1336.2.0.?
32064.t1 32064.t \( 2^{6} \cdot 3 \cdot 167 \) $1$ $\mathsf{trivial}$ $1.039758452$ $[0, 1, 0, -1185, 15327]$ \(y^2=x^3+x^2-1185x+15327\) 1336.2.0.?
96192.g1 96192.g \( 2^{6} \cdot 3^{2} \cdot 167 \) $2$ $\mathsf{trivial}$ $1.243978904$ $[0, 0, 0, -10668, 424496]$ \(y^2=x^3-10668x+424496\) 1336.2.0.?
96192.j1 96192.j \( 2^{6} \cdot 3^{2} \cdot 167 \) $0$ $\mathsf{trivial}$ $1$ $[0, 0, 0, -10668, -424496]$ \(y^2=x^3-10668x-424496\) 1336.2.0.?
100200.m1 100200.m \( 2^{3} \cdot 3 \cdot 5^{2} \cdot 167 \) $0$ $\mathsf{trivial}$ $1$ $[0, -1, 0, -7408, -243188]$ \(y^2=x^3-x^2-7408x-243188\) 1336.2.0.?
196392.l1 196392.l \( 2^{3} \cdot 3 \cdot 7^{2} \cdot 167 \) $0$ $\mathsf{trivial}$ $1$ $[0, -1, 0, -14520, 678924]$ \(y^2=x^3-x^2-14520x+678924\) 1336.2.0.?
200400.cf1 200400.cf \( 2^{4} \cdot 3 \cdot 5^{2} \cdot 167 \) $1$ $\mathsf{trivial}$ $2.043447180$ $[0, 1, 0, -7408, 243188]$ \(y^2=x^3+x^2-7408x+243188\) 1336.2.0.?
300600.bn1 300600.bn \( 2^{3} \cdot 3^{2} \cdot 5^{2} \cdot 167 \) $1$ $\mathsf{trivial}$ $6.870891690$ $[0, 0, 0, -66675, 6632750]$ \(y^2=x^3-66675x+6632750\) 1336.2.0.?
392784.co1 392784.co \( 2^{4} \cdot 3 \cdot 7^{2} \cdot 167 \) $0$ $\mathsf{trivial}$ $1$ $[0, 1, 0, -14520, -678924]$ \(y^2=x^3+x^2-14520x-678924\) 1336.2.0.?
484968.j1 484968.j \( 2^{3} \cdot 3 \cdot 11^{2} \cdot 167 \) $0$ $\mathsf{trivial}$ $1$ $[0, 1, 0, -35856, 2603808]$ \(y^2=x^3+x^2-35856x+2603808\) 1336.2.0.?
  displayed columns for results